Difference between revisions of "2016 AMC 10B Problems/Problem 2"

(Problem)
(Solution)
Line 9: Line 9:
 
==Solution==
 
==Solution==
 
<math>\textbf{(B)}\ \frac{1}{2}</math>
 
<math>\textbf{(B)}\ \frac{1}{2}</math>
 +
 +
Solution by ngeorge

Revision as of 10:25, 21 February 2016

Problem

If $n\heartsuit m=n^3m^2$, what is $\frac{2\heartsuit 4}{4\heartsuit 2}$?

$\textbf{(A)}\ \frac{1}{4}\qquad\textbf{(B)}\ \frac{1}{2}\qquad\textbf{(C)}\ 1\qquad\textbf{(D)}\ 2\qquad\textbf{(E)}\ 4$


Solution

$\textbf{(B)}\ \frac{1}{2}$

Solution by ngeorge